miércoles, 11 de enero de 2012

Problema del día

¡Hola a todos! Les dejo el primer problema del día de Geometría.

Sea $ABC$ un triángulo y $A_1$ el punto de contacto del excírculo correspondiente a $A$ con $BC$. Denotemos por $a$ a la recta que pasa por $A_1$ y es paralela a la bisectriz de $\angle BAC$. Definimos $b$ y $c$ de manera similar. Demuestra que $a, b$ y $c$ concurren. 

36 comentarios:

Unknown dijo...

¿Cómo van? ¿Qué han intentado?

Unknown dijo...

Yo intente algo de polos, ejes radicales... unas cositas de armónicos y hasta ceva trigonométrico pero nada :S

Adán dijo...

Estaba intentando probar que los polos de $a, b, c$ con respecto a la circunferencia inscrita a $ABC$ eran colineales. También intenté ver a $ABC$ como el triángulo órtico de los excentros de $ABC$, y ver si cumplían algo formando paralelogramos y cosas así.

Adán dijo...

Bueno, también intenté inversión con la circunferencia inscrita a $ABC$, pero tampoco...

Marco dijo...

Yo agarré un excírculo como circunferencia de inversión, encontré los polos de a,b y c y ,claro, quiero demostrar que son colineales, pero el problema se ve igual de complicado que al principio, aunque están relativamente fáciles de encontrar los polos, a diferencia de cuando agarras el incírculo como circ de inversión.
Más bien me late que tiene que ver con que las cevianas a A1 B1 y C1 concurren, y que las bisectrices también... pero la mera verdad ya me quedé sin ideas de ataque :/

Chuck dijo...

Yo intenté un poco de homotecia y de traslaciones. Luego usé un poco de cálculos con Ceva y el Teorema de la bicectriz, pero no servía. Ahora estoy intentando a ver si con complejos sale rápido. @Marco, eso no necesariamente es cierto, de hecho, según yo sólo se puede con dos parejas de puntos diferentes puntos en BC y AC para cada punto en AB. O al menos eso conjeturo con Regla y Compás

Juan dijo...

Según yo sale con Ceva trigonométrico pero serían muchas cuentas. Además de eso he visto ángulos y he intentado demostrar la concurrencia demostrando que los puntos de intersecciones eran harmónicos con algo, además de intentar algunas rotohomotecias y otras transformaciones para estudiar las líneas a, b, y c.

Marco dijo...

a que te refieres Chuck???

Chuck dijo...

A que no concurren solo por el hecho de que AA1, BB1 y CC1 concurran también.

Unknown dijo...

Bueno, pues de entrada me parece buena idea la de ver a ABC como el triángulo órtico del triángulo formado por los excentros. Sigan intentando un poco más, si de plano no sale nada, mañana les doy otra sugerencia.

Unknown dijo...

Renombraremos $a,b,c$ respectivamente como $l,m,n$. Consideremos coordenadas homogéneas(baricentricas) $A=(1,0,0),B=(0,1,0),C=(0,0,1)$. Si los lados $AB=c,BC=a,CA=b$ entonces $I=(a,b,c)$ es el incentro y $A_1=(0,s-b,s-c)$. Tenemos que la linea $l$ esta dada por, dada que es paralela a la recta $AI$ y pasa por $A_1$, (multiplicando desde antes por $a+b+c$ por el punto $I$ y por $a$ por el $A_1$)

$a \left|
\begin{array}{ccc}
x & 1 & a \\
y & 0 & b \\
z & 0 & c \\
\end{array}
\right|=\left|
\begin{array}{ccc}
0 & 1 & a \\
s-b & 0 & b \\
s-c & 0 & c \\
\end{array}
\right|(x+y+z)$

Entonces expandiendo y pasando a la izquierda tenemos que
$l:(sb-sc)x+(sb-sc+ac)y+(sb-sc-ab)z=0$
Similarmente
$m:(sc-sa-bc)x+(sc-sa)y+(sc-sa+ab)z=0$
y
$n:(sa-sb+bc)x+(sa-sb-ac)y+(sa-sb)z=0$.
Entonces como $l+m+n=0$ tenemos que si dos son $0$ la tercera también. Asi que se intersectan las tres lineas. QED

Unknown dijo...

Diego, explica mejor tu solución. ¿Lo último que pusiste se supone que son las ecuaciones de $l,m$ y $n$? Si es eso, está mal, esas son las ecuaciones de planos.

Unknown dijo...

Lo hago en coordenadas homogeneas, donde $(x,y,z)=(kx,ky,kz)$. Por eso son lineas y no planos.

Juan dijo...
Este comentario ha sido eliminado por el autor.
jorge garza vargas dijo...

Bonito resultado! aunque mi prueba no me gustó mucho.
Fijándose en el triángulo formado por los excentros es fácil ver que un problema equivalente es:
Sea $ABC$ un triángulo y $D,E,F$ los pies de altura desde $A, B, C$ respectivamente, sean $P,Q,R$ las proyecciones de $A,B,C$ a $EF, FD, DE$ respectivamente, entonces las perpendiculares a$BC, AC, AB$ por $P,Q,R$ respectivamente son concurrentes.
Lema 1. Es un hecho conocido que si $AB$ es un segmento, $P$ un punto y $X$ la proyección de $P$ sobre $AB$ entonces $AP^2-PB^2=AX^2-XB^2$.
Lema 2. Es un hecho conocido que si $ABC$ es un triángulo y $D,E,F$ puntos en $BC, CA, AB$ respectivamente, entonces las perpendiculares a los respectivos lados por $D,E,F$ concurren si y solo si $AF^2-FB^2+BD^2-DC^2+CE^2-EA^2=0$

jorge garza vargas dijo...

Luego, abordando el problema como mencioné al principio, y combinando los lemas 1 y 2, tenemos que las perpendiculares por $P,Q,R$ a los respectivos lados concurren si y solo si $\sum_{cyc}CP^2-PB^2=0$, luego llamando $x=\angle A, y=\angle B, z=\angle C$, luego por la ley de cosenos en el $APC$ tenemos que $CP^2=AP^2+AC^2-2sen(y)$, aplicando lo mismo para los segmentos de interés tenemos que la suma apenas mencionada sucede $\iff$, $\sum_{cyc} (AP^2+AC^2-2sen(y)\cdot AC\cdot AP)-(AB^2+AP^2-2sen(z)\cdot AP\cdot AB)=0$

jorge garza vargas dijo...
Este comentario ha sido eliminado por el autor.
jorge garza vargas dijo...

$\iff$ $(\sum_{cyc} AC^2-AB^2)$$+\sum{cyc} 2sen(z)\cdot AB\cdot AP-2sen(y)\cdot AC \cdot AP=0$, ahora definamos $P_A$ a la potencia de $A$ respecto a la circunferencia de diámetro $BC$, analogamente se definen $P_B, P_C$, luego, $sen(y)\cdot AC \cdot AP=sen^2(y)\cdot AE\cdot AC$$=sen^2(y)\cdot P_A$, por lo que el problema se reduce a demostrar que $\sum_{cyc} P_A(sen^2(y)-sen^2(z)=0)$, sea $a=BC, b=AC, c=AB$ y sean $h_a, h_b, h_c$ las alturas del triángulo, entonces nuestra suma se vuelve equivalente a $\sum{cyc} P_A(\frac{h_{c}^2-h_{b}^2}{a^2})=0$, luego si $M$ es el punto medio de $BC$ tenemos por definición de potencia que $P_A=AM^2-\frac{a^2}{4}$

jorge garza vargas dijo...

De finamos ahora $r=\frac{1}{2}(\sum_{cyc} a^2$. Usando la fórmula para poner la mediana en términos de los lados de un triángulo obtenemos $P_A=r-a^2$ y substituyendo lo obtenido en la suma que queríamos demostrar tenemos que queremos ver que $\sum_{cyc} (\frac{r}{a^2}-1)(h_{b}^2-h_{c}^2)=0$$=r(\sum_{cyc} \frac{h_{b}^2-h_{c}^2}{a^2}$$\iff$$\sum_{cyc} \frac{h_{b}^2}{a^2}=\sum{cyc} \frac{h_c^2}{a^2}$ y multiplicando ambos lados por $(abc)^2$ la igualdad se vuelve clara, con lo que hemos concluido la demostración.

alberto dijo...

No me salio, intente ver si ese punto era colineal con el incentro y la interseccion de $AA_1, BB_1, CC_1$ pero al parecer no son, y ceva trigonometrico pero no se me ocurre como avanzar. Que paso con la sugerencia? jeje

Unknown dijo...

La sugerencia es usar el Teorema de Carnot. ¿Todos lo conocen?

Adán dijo...

Sean $X, Y, Z$ los excentros opuestos a los vértices $A, B, C$ respectivamente. Sean $D, E, F$ las proyecciones de $X, Y, Z$ sobre $BC, CA, AB$ respectivamente. Como el incentro de $ABC$ es el ortocentro de $XYZ$, tenemos que las líneas que queremos resultan ser la paralelas a $XD, YE, ZF$ que pasan por $D, E, F$ respectivamente, pues $XD, YE, ZF$ son bisectrices de $ABC$. Ahora, por el teorema de Carnot y como nuestras $3$ rectas son perpendiculares a los lados de $XYZ$, tenemos que van a concurrir las $3$ rectas si y sólo si

$YD^{2}-ZD^{2}+ZE^{2}-XE^{2}+XF^{2}-YF^{2}$.

Ahora, definiremos los ángulos en base al ortocentro $H$ de $XYZ$, de el siguiente modo. Sean $\angle YXH=\angle ZXH=\alpha$, $\angle ZYH=\angle XYH=\beta$ y $\angle XZH=YZH=\gamma$. Por construcción sabemos que $\alpha+\beta+\gamma=90^{\circ}$ y además las líneas que van desde $X, Y, Z$ al circuncentro de $XYZ$ son isogonales a las alturas de $XYZ$, y van a voltear la manera en que los ángulos de $XYZ$ quedan partidos. Además, sabemos que por la construcción de $XYZ$, el mismo triángulo es acutángulo.

Ahora, podemos aplicar ley de cosenos en triángulo $XZF$ y obtenemos

$XF^{2}=XZ^{2}+FZ^{2}-2\cdot XZ\cdot FZ \cdot \cos{\alpha}$

y similarmente definimos los $6$ segmentos mencionados arriba.

Adán dijo...

Notemos que si $D_{Y}, D_{Z}, E_{Z}, E_{X}, F_{X}, F_{Y}$ son las proyecciones de $D$ sobre las $XY, XZ$; de $E$ sobre $YZ, YX$ y de $F$ sobre $ZX, ZY$ respectivamente tendremos que por la isogonalidad de las alturas y las líneas que van al circuncentro que mencionamos anteriormente, tendremos que

$DX\cos{\beta}=XD_{Z}$
$DX\cos{\gamma}=XD_{Y}$
$EY\cos{\gamma}=YE_{Z}$
$EY\cos{\alpha}=YE_{X}$
$FZ\cos{\alpha}=ZF_{Y}$
$FZ\cos{\beta}=ZF_{X}$

Adán dijo...

Entonces queremos que

$\sum_{cyc}{XY\cdot XF_{Y}-XZ\cdot XF_{Z}}=0$.

Adán dijo...

Entonces veamos que $XD_{Y}DD_{Z}$, $YE_{Z}EE_{X}$ y $ZF_{X}FF_{Y}$ son cuadriláteros cíclicos, pues definimos a nuestros vértices con subíndice como proyecciones. Como también los cuadriláteros $XYAB$, $YZBC$ y $ZXCA$ son cíclicos tendremos que

$\angle ZXY=\angle ZAB=\angle ZFF_{Y}=\angle ZF_{X}F_{Y}$

y haciendo esto análogamente a los otros $2$ pares de cíclicos veremos que $D_{Y}D_{Z}, E_{Z}E_{X}, F_{X}F_{Y}$ son paralelas a $YZ, ZX, XY$, y como $XYAB$, $YZBC$ y $ZXCA$ son cíclicos, por antiparalelas tendremos que $D_{Y}D_{Z}BC$, $E_{Z}E_{X}CA$ y $F_{X}F_{Y}AB$ son cíclicos.

Adán dijo...

Ahora, sean $YZ=a$, $ZX=b$ y $XY=c$. Entonces podemos ver, usando el hecho de que un triángulo recto de hipotenusa $x$ y uno de sus ángulos es $\theta$ tiene catetos de longitud $x\cos{\theta}, x\sin{\theta}$, que $BC=a\sin{\alpha}$, $CA=b\sin{\beta}$ y $AB=c\sin{\gamma}$.

Adán dijo...

Bueno, notemos las siguientes propiedades:

$1.$ Tenemos que $2$ veces el área de $XYZ$ la podemos expresar como

$bc\cos{\alpha}=ca\cos{\beta}=ab\cos{\gamma}$

Ahora, también veamos que

$a=b\cos{\gamma}+c\cos{\beta}$
$b=c\cos{\alpha}+a\cos{\gamma}$
$c=a\cos{\beta}+c\cos{\alpha}$

Esto aplicando la propiedad que mencioné sobre la hipotenusa de longitud $x$ y un ángulo $\theta$.

Adán dijo...
Este comentario ha sido eliminado por el autor.
Adán dijo...

Ahora, usando también la propiedad de la hipotenusa de longitud $x$ y el ángulo $\theta$ podemos ver que, por ejemplo

$F_{Y}Z=FZ\cdot \cos{\alpha}=AZ\cdot \left(\cos{\alpha}\right)^{2}$

pero como $XYZ$ es semejante con $BCX, CAY, ABZ$, tenemos que $AZ=b\sin{\gamma}$ y obtenemos que

$F_{Y}Z\cdot YZ=ab\sin{\gamma}\cdot \left(\cos{\alpha}\right)^{2}$

y los demás segmentos se van a definir de manera similar. En particular tenemos que lo que queremos es

$\sum_{cyc}{XY\cdot XD_{Y}-XZ\cdot XD_{Z}}$

que es lo que queremos que sea $0$, es igual a

$\sum_{cyc}{ab\sin{\gamma}\left(\cos{\alpha}^{2}-\cos{\beta}^{2}}$

Adán dijo...

Sea $R$ el circunradio de $XYZ$. Sabemos que $\sin{\gamma}=\frac{c\sin{\gamma}}{c}=\frac{c}{2R}$ por ley de senos. Entonces queremos que

$\sum_{cyc}{\frac{abc}{2R}\cdot\left(\cos{\alpha}^{2}-\cos{\beta}^{2}\right)}$

sea $0$, lo cual ahora es claramente cierto, pues cada coseno cuadrado se va a cancelar en la resta, y por lo tanto las $3$ rectas que queremos concurren.

Adán dijo...

NOTA: Arriba hay una suma cíclica que debe decir $D_{Y}, D_{Z}$ en vez de $F_{Y}, F_{Z}$. Gracias.

Enrique dijo...

Sean $D,E,F$ los excentros opuestos a $A,B,C$, resp., $P,Q,R$ los pies de las perpendiculares de $D,E,F$ a $BC,CA,AB$, resp., y $J,K,L$ los pies de las perpendiculares de $P,Q,R$ a $EF,FD,DE$, resp. Sean $\angle EDF=\alpha,\angle DEF=\beta, \angle EFD=\gamma$. Como $CBEF, BAED, ACDF$ son cíclicos, podemos llegar fácilmente a que $\angle QEA=\angle RFA=90-\alpha$, $\angle RFB=\angle PDB= 90-\beta$, $\angle PDC= \angle QEC=90-\gamma$. Es un hecho conocido que $ABC$ es triángulo órtico de $DEF$, por lo que debemos demostrar que $PJ,QK,RL$ concurren, y es un hecho conocido que esto pasa si y sólo si $EJ^{2}-JF^{2}+FK^{2}-KD^{2}+DL^{2}-LE^{2}=0$, que es equivalente a probar que $EP^{2}-FP^{2}+FQ^{2}-DQ^{2}+DR^{2}-ER^{2}=0$, pues por el lema de la perpendicular, $EJ^{2}-JF^{2}=EP^{2}-FP^{2}$ (esto es análogo para los 3 lados). Aplicando ley de cosenos sobre $DPE$, tenemos que $EP^{2}=DE^{2}+DP^{2}-2DE\cdot DP\cos (90-\gamma)$. Aplicando lo mismo para los segmentos $EP,FP,FQ,DQ,DR,ER$ con los triángulos $DPF,EQF,EQD,FRD,FRE$, resp., sustituyendo los valores de los segmentos en la ecuación y simplificando nos queda probar que $2(DP\cdot DE\cos (90-\gamma)-DP\cdot DF\cos (90-\beta)+$ $EQ\cdot EF\cos (90-\alpha)-EQ\cdot ED\cos (90-\gamma)+$ $FR\cdot FD\cos (90-\beta)-FR\cdot FE\cos (90-\alpha))=0$. Ahora, como $DCB$ y $DEF$ son semejantes, $DP=\frac{DB\cdot DA}{DE}=\frac{DC\cdot DA}{DF}$. Análogamente, $EQ=\frac{EC\cdot EB}{EF}=\frac{EA\cdot EB}{ED}$ y $FR=\frac{FB\cdot FC}{FE}=\frac{FA\cdot FC}{FD}$. Sustituyendo $DP,EQ,FR$ en la ecuacion y simplificando nos queda que debemos demostrar que $2(DB\cdot DA\cos (90-\gamma)-DC\cdot DA\cos (90-\beta)+$ $EC\cdot EB\cos (90-\alpha)-EA\cdot EB\cos (90-\gamma)+$ $FA\cdot FC\cos (90-\beta)-FB\cdot FC\cos (90-\alpha))=0$. Tenemos que, por ley de cosenos sobre $DBA$, $2DB\cdot DA\cos (90-\gamma)=DB^{2}+DA^{2}-AB^{2}$. Haciendo esto para todos los sumandos con los triángulos $DCA,EBC,EBA,FCA,FCB$, sustiuyendo cada sumando de la ecuación por el equivalente que nos dio la ley de cosenos y simplificando nos queda demostrar que $DB^{2}-BF^{2}+FA^{2}-AE^{2}+EC^{2}-CD^{2}$, lo cual es evidentemente cierto. QED

Juan dijo...

Ya me salió pero está feo. Básicamente modifico el problema para que diga (usando el triangulo de excentros): Si ABC es un triángulo y DEF su triángulo órtico, y X, Y y X las proyecciones de A, B y C a FE, FD, y ED respectivamente, y las proyecciones de X, Y y Z a BC, CA y AB respectivamente son K, L y M, entonces demostrar que XK, YL y ZM concurren.

Para demostrar eso, basta con calcular CK y KB (y análogamente calcularíamos AM. MB, LA, LC), y usando el Teorema de Carnot acabaríamos. Para ésto, digamos que O es la intersección de AZ con BC. Entonces podemos usar Tales para conseguir OK = OD*OQ/OA, donde Q es el circuncentro del triángulo (usando la isogonalidad de Q y el ortocentro). OA se puede conseguir con la Ley de Senos en AOC, con OA/senC = AC/senAOC, y sabemos AOC, pues es 180°-ACB-OAC = 180-ACB-(90-ABC)=
90-C+B. Así calculamos OA.
Para calcular OQ, basta ver que OQ=OA-R, donde R es el circunradio, que es BC/2senA por ley de Senos.
Finalmente, para calcular OD, calculamos BD con trigonometría (AD es altura), luego OC con Ley de Senos otra vez en AOC (OC=ACsenAOC/senOAC, y calculamos los ángulos usando la isogonalidad de Q y el otrocentro, como calculamos AOC), y luego como tenemos BC podemos caluclar DO=BC-BD-OC.

Así, ya con nuestros 3 valores, podremos calcular OK. Entonces KC será OK+OC y OC ya lo sabíamos por la ley de senos en AOC, y tendremos que BK=CB-CK. Así, habremos calculado BK y CK, análogamente AL,LC,AM,BM y acabamos por el teorema de Carnot (KX, LY y MZ son perpendiculares BC, AC y CA respectivamente, y L, M y K están sobre los lados del triángulo. Pero las cuentas no están anda bonitas.

JulioC dijo...

Sean D,E,F, los puntos de interseccion de los excirculos de ABC con BC, CA, AB, resp. Sean G,H, I, los centros de los excirculos correspondientes a A, B, C resp. Es conocido que ABC es el triagulo ortico de GHI y que las bisectrices de ABC son las alturas de GHI. Por lo tanto a, b y c son las perpendiculares desde D, E y F a HI, GI, HG, resp. Entonces por el teorema de Carnot a,b,c son concurrentes si y solo si
DI^{2} + EG^{2} + FH^{2} = DH^{2} + EI^{2} + FG^{2}
Sean r1, r2 y r3 los radios de los excirculos con centro G, H, I, resp.
Si X es el punto de interseccion del excirculo con I y BC. Entonces IX es perpendicular a BC, entonces por Pitagoras DI^{2}= IX^{2} + DX^{2} = r3^{2} + DX^{2}
Pero es conocido que si a,b,c son los las longitudes de BC, CA, AB, resp. Y s el semiperimetro de a,b,c. Entonces es conocido que DB= s-c y FB= BX= s-a. Entonces DX= DB + FB = s-c +s-a = b.
Entonces DI^{2}= r3^{2} + b^{2}. Analogamente con todas las demas longitudes tenemos.
DI^{2} + EG^{2} + FH^{2} = r1^{2} + r2^{2} + r3^{2} + a^{2} + b^{2} + c^{2} =DH^{2} + EI^{2} + FG^{2}
Por lo tanto a,b,c concurren

Unknown dijo...
Este comentario ha sido eliminado por el autor.
Unknown dijo...

Primero, renombré a $\A_1,B_1, C_1 $ como $\ P, Q, R $ respectivamente, y llamé $\ D, E, F $ a los excentros opuestos a $\ A, B, C $ respectivamente.

Además sean $\ M, N, L $ las intersecciones de las rectas $\ a, b y c $ con $\ EF, FD, DE $.

Sean $\ \angle ABC = 2\beta, \angle BCA = 2\gamma, \angle BAC = 2\alpha $

Es un resultado conocido que el punto de tangencia de los excentro en cada lado y el putn ode tangencia del incentro son isotómicos, de donde tenemos:
$\ AR=PC=z, AQ=BP=x BR=CQ=y $

Notamos que $\ ABC $ es el triángulo órtico de $\ DEF $ y que $\ I $ es su ortocentro.
Como $\ a, b ,c $ son paralelas a las bisctrices (que son alturas) tenemos que son perpendiculares a los lados de $\ DEF $.

Usando lo anterior y por el teorema de Carnot, vemos que $\ a, b ,c $ concurren si y sólo si:
$$\ FM^2-ME^2 + EL^2-LD^2 + DN^2-NF^2 = 0 $$.

Pero aplicando el lema de perpendicularidad, esto equivale a:
$$\ FP^2-PE^2 + ER^2-RD^2 + DQ^2-QF^2 = 0 $$. (1)

Por ley de cosenos tenemos que $\ FP^2 = x^2 + FB^2 - 2\cdotx\cdotFB\cdotCOS(\angle FBP) $
Pero como las bisectrices internas y externas son perpendiculares, tenemos que $\ cos (\angle FBP) = -sen \beta $
y además podemos escribir también que $\ FB= \frac{y}{sen \beta} $
Haciendo las sustituciones correspondientes tenemos que:

$$\ FP^2= FB^2+x^2+2xy $$

Podemos hacer un proceso similar con cada elemento en (1) y vemos con ello que (1) equivale a que:
$\ a, b ,c $ concurren si y sólo si:
$$\ FA^2-AE^2+EC^2-CD^2+BD^2-FB^2 = O $$
pero como estos son los pies de las alturas del triángulo $\ DEF $ por teorema de Carnot nuevamente, esto es cierto, con lo que damos por concluido el problema.

Publicar un comentario